11. a $58 video game is on sale for a 14% discount. what is the price with the discount

Answers

Answer 1

As given by the question

There are given that the price of the video games is $58.

Now,

According to the question, there are given that 14% discount on their game price

So,

The price with discount is:

[tex]58\times\frac{14}{100}=8.12[/tex]

Then,

[tex]58-8.12=49.88[/tex]

Hence, the price with the discount is $49.88


Related Questions

Plssss help!
Geometry A
Brainliest

Answers

Answer:

18 ft.

Step-by-step explanation:

The function f(x) = 4x − 6 is shown in the table below. Identify the domain and range of function f. Enter the numbers in order from least to greatest.

x −5 −2 1 5
y −26 −14 −2 −14

Answers

The domain and the range of the function are {−5, −2, 1 ,5} and {−26, −14, −2} respectively

How to determine the domain and the range?

From the question, we have the following parameters that can be used in our computation:

Function: f(x) = 4x - 6

Table of values

x −5 −2 1 5

y −26 −14 −2 −14

The set of x values is the domain

So, we have

Domain = {−5 −2 1 5}

Rewrite as

Domain = {−5, −2, 1 ,5}

The set of y values is the range

So, we have

Range = {−26 −14 −2 −14}

Rewrite as

Range = {−26, −14, −2}

Hence, the range is {−26, −14, −2}

Read more about domain and range at

https://brainly.com/question/2264373

#SPJ1

Find the value of x in the triangle shown below

Answers

The answer is C, the square root of 65 (8.062).

Solve the system using the substitution technique:(−6, −3.6)(0.6, 0.8)(6, 4.4)(−0.6, 0)

Answers

Given

The system of equations,

[tex]\begin{gathered} -2x+3y=1.2\text{ \_\_\_\_\_}(1) \\ -3x-6y=1.8\text{ \_\_\_\_\_}(2) \end{gathered}[/tex]

To find the solution using substitution technique.

Explanation:

It is given that,

[tex]\begin{gathered} -2x+3y=1.2\text{ \_\_\_\_\_}(1) \\ -3x-6y=1.8\text{ \_\_\_\_\_}(2) \end{gathered}[/tex]

That implies,

[tex]\begin{gathered} (1)\Rightarrow-2x+3y=1.2 \\ \Rightarrow3y=1.2+2x \end{gathered}[/tex]

And,

[tex]\begin{gathered} (2)\Rightarrow-3x-6y=1.8 \\ \Rightarrow-3x-2(3y)=1.8 \end{gathered}[/tex]

Substitute 3y=1.2+2x in the above equation.

That implies,

[tex]\begin{gathered} -3x-2(1.2+2x)=1.8 \\ -3x-2.4-4x=1.8 \\ -7x=1.8+2.4 \\ -7x=4.2 \\ x=\frac{4.2}{-7} \\ x=-0.6 \end{gathered}[/tex]

And, substitute x=-0.6 in (1).

That implies,

[tex]\begin{gathered} (1)\Rightarrow-2(-0.6)+3y=1.2 \\ \Rightarrow1.2+3y=1.2 \\ \Rightarrow3y=1.2-1.2 \\ \Rightarrow3y=0 \\ \Rightarrow y=0 \end{gathered}[/tex]

Hence, the solution is (-0.6,0).

Follow the steps to solve the equation.

Answers

Given equation is

[tex] \sqrt[3]{x {}^{2} - 7 } = \sqrt[3]{2x + 1} [/tex]

to solve this equation we first need to cube both the sides . As this would remove the cube root on both the sides ,

[tex]\longrightarrow (\sqrt[3]{x^2-7})^3 = (\sqrt[3]{2x+1})^3[/tex]

This would become ,

[tex]\longrightarrow x^2-7=2x+1 \\[/tex]

[tex]\longrightarrow x^2-2x-7-1=0\\[/tex]

[tex]\longrightarrow x^2-2x-8=0\\[/tex]

[tex]\longrightarrow x^2 -4x +2x -8=0\\[/tex]

[tex]\longrightarrow x(x-4)+2(x-4)=0\\ [/tex]

[tex]\longrightarrow (x+2)(x-4)=0\\[/tex]

[tex]\longrightarrow \underline{\underline{ x = 4,-2}} [/tex]

And we are done!

Answer:

[tex]\textsf{To solve the given equation, $\boxed{\sf cube}$ both sides}.[/tex]

Step-by-step explanation:

Given equation:

[tex]\sqrt[3]{x^2-7} =\sqrt[3]{2x+1}[/tex]

[tex]\textsf{Apply exponent rule} \quad \sqrt[n]{a}=a^{\frac{1}{n}}:[/tex]

[tex]\implies (x^2-7)^{\frac{1}{3}}=(2x+1)^{\frac{1}{3}}[/tex]

Cube both sides of the equation:

[tex]\implies \left( (x^2-7)^{\frac{1}{3}}\right)^3= \left((2x+1)^{\frac{1}{3}}\right)^3[/tex]

[tex]\textsf{Apply exponent rule} \quad (a^b)^c=a^{bc}:[/tex]

[tex]\implies (x^2-7)^{\frac{3}{3}}=(2x+1)^{\frac{3}{3}}[/tex]

[tex]\implies (x^2-7)^{1}=(2x+1)^{1}[/tex]

[tex]\textsf{Apply exponent rule} \quad a^1=a:[/tex]

[tex]\implies x^2-7=2x+1[/tex]

Subtract 2x from both sides:

[tex]\implies x^2-2x-7=1[/tex]

Subtract 1 from both sides:

[tex]\implies x^2-2x-8=0[/tex]

Rewrite -2x as (-4x + 2x):

[tex]\implies x^2-4x+2x-8=0[/tex]

Factor the first two terms and the last two terms separately:

[tex]\implies x(x-4)+2(x-4)=0[/tex]

Therefore:

[tex]\implies (x+2)(x-4)=0[/tex]

Apply the zero-product property:

[tex]x+2=0 \implies x=-2[/tex]

[tex]x-4=0 \implies x=4[/tex]

SOMEONE PLEASE ANSWER THIS QUESTION!

Answers

The answer would be the second one 5^2+4

What is the slope of the line that passes through the points (5, 8) and (11, 5)? Write
your answer in simplest form.

Answers

m = 0
Step-by-step explanation:
Let P2 = (-11, 8)
P1 = (-5, 8)
Use the following equation for the slope:

= (8 - 8) / (-5 + 11)
= 0
This means that the line is horizontal.

Midpoint: 8, -2 Endpoint: 3,10 find other point

Answers

The midpoint coordinates are: (8,-2)

We can label this coordinates as follows

[tex]\begin{gathered} x_m=8 \\ y_m=-2 \end{gathered}[/tex]

The coordinates for one of the endpoints are: (3,10)

We label this coordinates as follows:

[tex]\begin{gathered} x_1=3 \\ y_1=10_{}_{} \end{gathered}[/tex]

We are looking for the other endpoints with the coordinates (x2,y2).

We use the formulas to find the midpoint:

[tex]\begin{gathered} x_m=\frac{x_1+x_2}{2}_{} \\ y_m=\frac{y_1+y_2}{2} \end{gathered}[/tex]

But, since we need x2 and y2, we solve for then in the equations:

[tex]\begin{gathered} 2x_m-x_1=x_2_{} \\ 2y_m-y_1=y_2 \end{gathered}[/tex]

And we substitute our values into the equations:

For x2:

[tex]\begin{gathered} 2(8)-3=x_2 \\ 16-3=x_2 \\ 13=x_2 \end{gathered}[/tex]

For y2:

[tex]\begin{gathered} 2(-2)-10=y_2 \\ -4-10=y_2 \\ -14=y_2 \end{gathered}[/tex]

Answer: (13,-14)

110.4 ÷4 step bye step show me pictures

Answers

we ahve

110.4 ÷4

so

110.4/4

Multiply by 10/10 the expression

1104/40

simplify

1104/40=552/20=276/10=27.6

the answer is 27.6

A sector with a radius \maroonD{18\,\text{cm}}18cmstart color #ca337c, 18, start text, c, m, end text, end color #ca337c has an area of \goldE{234\pi\,\text{cm}^2}234πcm 2 start color #a75a05, 234, pi, start text, c, m, end text, squared, end color #a75a05.

Answers

The formula for the area (A) of the sector is,

[tex]A=\frac{\theta}{360^0}\times\pi r^2[/tex]

Given

[tex]\begin{gathered} r=18cm \\ A=234\pi cm^2 \end{gathered}[/tex]

Therefore,

[tex]234\pi=\frac{\theta}{360}\times\pi(18)^2[/tex]

Solve for θ

[tex]\begin{gathered} \frac{θ}{360}\pi \left(18\right)^2=234\pi \\ \frac{9\pi θ}{10}=234\pi \\ \frac{10\times \:9\pi θ}{10}=10\times \:234\pi \\ 9\pi θ=2340\pi \\ \mathrm{Divide\:both\:sides\:by\:}9\pi \\ \frac{9\pi θ}{9\pi }=\frac{2340\pi }{9\pi } \\ \thereforeθ=260^0 \end{gathered}[/tex]

Hence, the answer is

[tex]260^0[/tex]

Between what two standard deviations of a normal distribution contain 95% of the data?

Answers

Approximately 95% of the data fall within 2 standard deviations of the mean. That is, 2 standard deviations below and 2 standard deviations above the mean as in the following picture:

What is the probability of either event occurring when you spina spinner with the numbers 1 through 4 which are all evenlyrepresented?Event A: Spinning an odd numberEvent B: Spinning a 4

Answers

We are given an experiment, and we are asked about the probability that either event happens. Therefore, we use the addition rule of probability, which states that if A and B are two events in a probability experiment the probability that either one of the events to happen is:

[tex]P(A\text{ }or\text{ }B)=P(A)+P(B)\text{ - }P(A\text{ }and\text{ }B)[/tex]

Therefore, in our specific case, we have that the probability of A is 1/2, since we have 2 odd numbers out of possible 4 outcomes. The probability of B is 1/4, since we have a number 4 out of 4 possible outcomes. The probability of A and B is 0, because obtaining a 4 and an odd number are two mutually exclusive events. Therefore we have that our probability simply is the sum of our two probabilities:

[tex]P(A\text{ }or\text{ }B)=\frac{1}{2}+\frac{1}{4}=\frac{2}{4}+\frac{1}{4}=\frac{3}{4}[/tex]

Therefore, our answer is 3/4

Write the equation of the line that passes through the points (9,-1)(9,−1) and (-7,4)(−7,4). Put your answer in fully simplified form, unless it is a vertical or horizontal line.

Answers

The equation for lines will be y = -5/16 x + 29/16

What is equation straight line?

Y = mx + c is the general equation for a straight line, where m denotes the line's slope and c the y-intercept. It is the version of the equation for a straight line that is used most frequently in geometry. There are numerous ways to express the equation of a straight line, including point-slope form, slope-intercept form, general form, standard form, etc. A straight line is a geometric object with two dimensions and infinite lengths at both ends. The formulas for the equation of a straight line that are most frequently employed are y = mx + c and axe + by = c. Other versions include point-slope, slope-intercept, standard, general, and others.

The equation of the right line is [tex]\frac{(y+1)}{(x-9)} =m \frac{(4+1)}{(-7-9)}[/tex]

[tex]\frac{(y+1)}{(x-9)} = \frac{(4+1)}{(-7-9)}[/tex]

[tex]\frac{(y+1)}{(x-9)} = -5/16[/tex]

16y+16 =-5x +45

16y = -5x +29

y = -5/16 x + 29/16

Hence the equation for lines will be y = -5/16 x + 29/16

Learn more about the straight line, by the following link

https://brainly.com/question/16949303

#SPJ1

If f(x) = x² +3,then f (x + h) =

Answers

The function is  f(x) = x² +3,then f (x + h)  = [tex]x^2+2xh +h^2+3[/tex].

Given,

In the question:

The function is given as:

f(x) = x² +3

To find the f (x + h) = ?

Now, According to the question:

Substitute x = x + h into f(x) = x² +3

f(x + h) =  (x+ h)² +3

Expand the expression using:

[tex](a +b)^2 = a^2 +2ab+b^2[/tex]

f(x + h) = [tex]x^2+2xh +h^2+3[/tex]

Hence, The function is  f(x) = x² +3,then f (x + h)  = [tex]x^2+2xh +h^2+3[/tex].

Learn more about Function at:

https://brainly.com/question/17403806

#SPJ1

Write these numbers in order of size, starting with the smallest?

0.45
4.5
0.045
0.405
4.05

Answers

Answer:

0.045, 0.405, 0.45, 4.05, 4.5

Step-by-step explanation:

So, we have 0.045 before 0.405 because the second number (0) in the first one is lower than the second number (4) in the second one. We have 0.405 before 0.45 because the third numbers don't match up. The lower one is 0, while the higher one is 5. Next, 4.05 is smaller than 4.5 because of the second number. The second number in the smaller one is 0, while the bigger one, its second number is 4.5.

May I have Brainliest please? My next rank will be the highest one: A GENIUS! Please help me on this journey to become top of the ranks! I would really appreciate it, and it would make my day! Thank you so much, and have a wonderful rest of your day!

12. Jayden sold 103 tickets for the school play. Student tickets cost $9 and adult tickets cost $14. Jayden's sales totaled $1127. How many adult tickets and how many student tickets did Jayden sell?​

Answers

The total number of student tickets which are sold is 63 and the total number of adult tickets which are sold is 40.

Let the students' tickets sold by Jayden be x. Then, the number of adult tickets which are sold be 103-x.

Now, one student ticket costs $9.

Total cost of student tickets which are being sold = $9x

Similarly, one adult ticket costs $14.

Total cost of adult tickets which are being sold = $14×(103-x)

Total sales done by Jayden = $1127

∵  $9x +  $14×(103-x) =  $1127

⇒ 9x + 14×(103-x) = 1127

⇒ 9x + 1442 - 14x = 1127

⇒ 14x - 9x = 1442 - 1127

⇒ 5x = 315

∴ x = 63

⇒ 103 - x = 103 - 63 = 40

Hence, the total number of student tickets which are sold is 63 and the total number of adult tickets which are sold is 40.

To find similar questions on 'tickets'

visit- https://brainly.com/question/24749333

#SPJ9

A store manager paid $44.00 for a shirt. She marked up the price of the shirt by 80% and then sold the shirt what was the selling price of the shirt

Answers

[tex]\text{selling price of shirt = \$79.2}[/tex]Explanation:

Cost price = $44

mark up percent = 80% = 80/100 = 0.8

selling price = ?

The markup formula:

[tex]markup=\text{ }\frac{selling\text{ price - cost price}}{\cos t\text{ price}}[/tex][tex]\begin{gathered} 0.8=\text{ }\frac{selling\text{ price - 4}4}{44} \\ \text{cross multiply:} \\ \text{0.8(44) }=\text{ selling price - 44} \\ \end{gathered}[/tex][tex]\begin{gathered} 35.2\text{ = selling price - 44 } \\ 35.2\text{ + 44 = selling price } \\ \text{selling price = \$79.2} \end{gathered}[/tex]

If you know that 84 is 70% of the whole, how can you use proportional reasoning to determine the whole?

70100 shows 70% as a ratio. The ratio 84x compares 84 to the whole, x. Write an equation so that the first ratio is equal to the second ratio. Solve for x : x = 120.

70100 shows 70% as a ratio. The ratio 84x compares 84 to the whole, x. Write an equation so that the first ratio is multiplied by the second ratio. Solve for x : x = 58.8.

70100 shows 70% as a ratio. The ratio x84 compares 84 to the whole, x. Write an equation so that the first ratio is multiplied by the second ratio. Solve for x : x = 120.

70100 shows 70% as a ratio. The ratio x84 compares 84 to the whole, x. Write an equation so that the first ratio is equal to the second ratio. Solve for x : x = 58.8.

Answers

The value of x is 120.

70/100 shows 70% as a ratio. The ratio x : 84 compares 84 to the whole, x. Write an equation so that the first ratio is multiplied by the second ratio. Solve for x : x = 120.

Given, that 84 is 70% of the whole.

Let the whole be x,

According to the question,

(70/100) × x = 84

On multiplying both the sides by 100/70, we get

x = 84×100/70

Now, on solving the expression, we get

x = 120

Hence, 70/100 shows 70% as a ratio. The ratio x : 84 compares 84 to the whole, x. Write an equation so that the first ratio is multiplied by the second ratio. Solve for x : x = 120.

Learn more about Percentages here https://brainly.com/question/24877689

#SPJ1

7. Compare Ann and Barry Lindale's expenses for renting versus owning a home inthe table below.a. Complete each table.b. Is it less expensive for them to buy or rent a home, and what is the difference?

Answers

To complete the tables, we multiply the given numbers:

First table:

[tex]\begin{gathered} Rent:\text{ 850}\times12=10200, \\ Phone,\text{ Internet \& Cable TV: }99\times12=1188. \end{gathered}[/tex]

Second table:

[tex]Phone,\text{ Internet \& Cable TV: }99\times12=1188.[/tex]

Now, to determine which option is less expensive we compute the total annual expenses for each case:

1.-Rental:

[tex]10200+45+180+1560+2400+1188=15,573.[/tex]

2.- Homeowner:

[tex]6600+3600+480+1710+2400+1188+570+960+1180=18,688.[/tex]

From the above calculations, we can conclude that the cheaper option based on the given annual expenses is to rent.

Answer:

Renting is less expensive.

in 2000, the total population of the u.s. was 281.4 million people. in 2010, it was 308.7 million people. (source: www.census.gov) what is the average rate of change in the total population over this time period?

Answers

The average rate of change is 2,73,000.

The average rate of change is calculated using the formula -

Average rate of change = change in population ÷ change in time

Keep the values in formula to find the rate of change of population over given time period

Average rate of change = (308.7 - 281.4) million ÷ (2010 - 2000)

Performing subtraction in numerator and denominator on Right Hand Side of the equation

Average rate of change = 2730000 ÷ 10

Performing division on Right Hand Side of the equation

Average rate of change = 2,73,000

Learn more about rate of change -

https://brainly.com/question/8728504

#SPJ4

The value of x equals the number of cubic units in a box that is 4 units high, 4 units deep, and 4 units wide. Which equation can be used to determine the value of x?Math item stem image3x=644x=64x−−√=4x−−√3=4

Answers

The value of x equals the number of cubic units in a box that is 4 units high, 4 units deep, and 4 units wide.

Recall that the volume of a cube is given by

[tex]V=l\cdot w\cdot h[/tex]

Where l is the length, w is the width, and h is the height of the cube.

We are given that all three sides are 4 units.

So, the volume is

[tex]\begin{gathered} V=4\cdot4\cdot4\; \\ V=64\; \; cubic\; \text{units} \end{gathered}[/tex]

x must be equal to this volume

[tex]x=64[/tex]

Take cube root on both sides of the equation

[tex]\begin{gathered} \sqrt[3]{x}=\sqrt[3]{64} \\ \sqrt[3]{x}=\sqrt[3]{4^3} \\ \sqrt[3]{x}=4 \end{gathered}[/tex]

Therefore, the correct equation is the last option.

[tex]\sqrt[3]{x}=4[/tex]

What is the volume of a hemisphere with radius 3 ft? What is the volume of a hemisphere with diameter 13 cm?

Answers

The volume of a sphere is given by

[tex]V_s=\frac{4}{3}\pi r^3[/tex]

Since a hemisphere is half sphere, its volume is given by

[tex]\begin{gathered} V_{}=\frac{4}{6}\pi r^3 \\ \text{which is equivalent to} \\ V_{}=\frac{2}{3}\pi r^3 \end{gathered}[/tex]

where r is the radius.

Case a.

In this part r=3 ft, then by substituting this values into our last formula we get

[tex]V=\frac{2}{3}(3.1416)(3^3)[/tex]

which gives

[tex]V=56.55ft^3[/tex]

Case b.

In this part r=(13/2) cm, then by substituting this values into our last formula we get

[tex]V=\frac{2}{3}(3.1416)(6.5^3)[/tex]

which gives

[tex]V=287.59cm^3[/tex]

Write the coefficient of x in the following terms
3. xy
1. 24x
4. x
2. abx

5. 3xy
6. xyz

Answers

Answer:

1. 1

2. 24

3. 1

4. 1 [ab]

5. 3

6. 1

Step-by-step explanation:

Every variable or alphabet's coefficient is the number on the left of it.

e. g coefficient of x is 1.

or coefficient of 83x is 83

write an exponential function to model the situation. find the amount after the specified time. $1,000 principal, 3.6% compounded monthly for 10 years

Answers

We can model this problem by an exponential growth:

[tex]A=P(1+\frac{r}{n})^{nt}[/tex]

where A is the amount accumulated, P is the principal, r is the interest rate, n is the number of times per year and t is the time. By substituting our given data, we get

[tex]\begin{gathered} A=1000(1+\frac{0.036}{12})^{12t} \\ A=1000(1+0.003)^{12t} \end{gathered}[/tex]

therefore, the model is

[tex]A=1000(1.003)^{12t}[/tex]

Now, by substituting t=10 years, we have

[tex]A=1000(1.003)^{120}[/tex]

then, the amount will be

[tex]A=1432.55\text{ dollars}[/tex]

(2x²+4-5x)-(-8x+2+x²) is a what kind of polynomial

Answers

Answer:

The degree of the polynomial is the greatest of its various terms' exponents (powers). This is a seconds degree polynomial.

Step-by-step explanation:

2x^2 - 3x^5 + 5x^6.

We observe that the above polynomial has three terms. Here the first term is 2x^2, the second term is -3x^5 and the third term is 5x^6.

Now we will determine the exponent of each term.

(i) the exponent of the first term 2x^2 = 2

(ii) the exponent of the second term 3x^5 = 5

(iii) the exponent of the third term 5x^6 = 6

Since the greatest exponent is 6, the degree of 2x^2 - 3x^5 + 5x^6 is also 6.

Therefore, the degree of the polynomial 2x^2 - 3x^5 + 5x^6 = 6.

Easy peasy, hope you understand now.

An = -8 - (n-1)2

what’s an equation equivalent
An = -2n-6
An = -2n-10
An = -8n -2
An = -2 - 8(1-n)

Answers

First one
Distribute negative and multiply= -8-2n+2
Reorder and combine like terms= -2n-6
Final Answer^

8−6÷2+3×5= 7×3−15÷5= 8+2(1+12÷2)^2=

Answers

Recall that the order of the operations PEMDAS

1. Parenthesis

2. Exponents.

3. Multiplication.

4. Division.

5. Addition.

6. Subtraction.

1. Given expression is

[tex]8-6\div2+3\times5[/tex]

Multiply 3 and 5, we get

[tex]8-6\div2+3\times5=8-6\div2+15[/tex]

Divide 6 by 2, we get

[tex]=8-3+15[/tex]

Subtract 3 from 8, we get

[tex]=5+15[/tex]

Add 5 and 15, we get

[tex]=20[/tex]

The answer is

[tex]8-6\div2+3\times5=20[/tex]

2.Given expression is

[tex]7\times3-15\div5[/tex]

Multiply 7 and 3, we get

[tex]7\times3-15\div5=21-15\div5[/tex]

Divide 15 by 5, we get

[tex]=21-3[/tex]

Subtract 3 from 21, we get

[tex]=18[/tex]

The answer is

[tex]7\times3-15\div5=18[/tex]

3. Given expression is

[tex]8+2(1+12\div2)^2[/tex]

First, we need to solve inside the parenthesis, divide 12 by 2, we get

[tex]8+2(1+12\div2)^2=8+2(1+6)^2[/tex]

Add 1 and 6, we get

[tex]=8+2(7)^2[/tex]

Solve exponent.

[tex]=8+2(49)[/tex]

Multiply 2 and 49, we get

[tex]=8+98[/tex]

Add 8 and 98, we get

[tex]=106[/tex]

The answer is

[tex]8+2(1+12\div2)^2=106[/tex]

A writer counted the number of pages she wrote in one year. The graph shows the relationship between the number of short stories written, x, and the number of pages written, y. coordinate plane with the x axis labeled number of short stories and the y axis labeled number of pages with a line that passes through the points 0 comma 1 and 1 comma 3 Part A: Calculate the slope of the linear equation shown in the graph. Show all necessary work. (3 points) Part B: What does the slope mean for the relationship between the number of pages written and the number of short stories written? (3 points) Part C: Interpret the y-intercept in the situation. (3 points) Part D: Write the equation of the line shown on the graph in slope-intercept form. (3 points)

Answers

I hope this helps! Good luck! Lmk if you need more help dude.

Step-by-step explanation:

A. Slope formula: y2-y1/x2-x1

(0,1) and (1,3)

3-1/1-0=2

The slope is 2.

B. I think the slope means there are 2 pages for every 1 short story written.

C. y=2x+1

2 is the slope, we see the line crosses through at (0,1) so it is our y-intercept.

A. The slope of the linear equation is 2.

B. The slope indicates that the number of pages written increases by a constant rate of 2 pages.

C. The y-intercept means that the initial number of page is 1.

D. The equation of the line in slope-intercept form is y = 2x + 1.

How to determine the slope and equation of this graph?

Part A. First of all, we would determine the slope of the line represented by this graph;

Slope (m) = (y₂ - y₁)/(x₂ - x₁)

Slope (m) = (7 - 3)/(3 - 1)

Slope (m) = 4/2 = 2.

Part B.

The meaning of the slope is that the number of pages written by this writer increases by 2 every year. Therefore, there are two number of pages for every (1) short story that is written by the writer.

Part C.

The y-intercept is located at point (0, 1) and in the context of the situation, it indicates that there is a page when the number of short story is equal to zero (0) or when there are no short stories written.

Part D.

In Mathematics and Geometry, the slope-intercept form of a straight line can be calculated by using the following mathematical equation:

y = mx + b

Where:

x and y represent the points.b is the y-intercept.m represent the slope.

By substitution, a linear equation for the line is given by:

y = mx + b

y = 2x + 1

Read more on slope-intercept here: brainly.com/question/7889446

#SPJ2

Using the counting principle determine the number of elements in the sample space. Two digits are selected without replacement from the digits 1,2,3,4,5 and 6

Answers

Consider the experiment of picking one digit first from the initial set (digits from 1 t) 6, and then pgrabbing a second digit without eplace,ment.

As for the first part of the experiment, there are 6 digits to choose from; however, during the second round, there are only 5 digits available. Therefore, according to the counting principle, the sample space has

[tex]6*5=30[/tex]

30 elements. he answer is 30 elements in the sampele space.

The equations in elimination method are written in the standard form as _______________.ax - by = cax + by + c = 0Ax + By = Cax - by - c = 0

Answers

Step 1

Given; The equations in the elimination method are written in the standard form as ____

Step 2

Simultaneous linear equations can be solved using the elimination method. First of all, make sure that the equations are written in the standard form either Ax+By=C or Ax+By+C=0. In this method, we multiply both the equations with a non-zero number to make the coefficients of any one variable equal.

Thus the answer is; The equations are written in standard form as seen below

[tex]Ax+By=C[/tex]

Other Questions
I need help on in the answer to this question a building has 63 apartments in 70 bathrooms what is the ratio Apartments to bathrooms in the building simplest form how the example dystopian work fits into the elements of dystopian literature Which answer below correctly identifies two partsof a function?1 arguments and executables2 arguments and statements3 statements and Python4 executables and programs John wants to fence a rectangular garden, which is 30 feet. What would be the dimensions of the fence enclosing the greatest area?Options a) 7.7 x 7.5 b) 8 x 7 c) 20 x 10d) 15 x 15 Question 7 of 9Constance has an apple that weighs 85,000 milligrams and a peach that weighs 0.15 kilograms. Which fruit has the greater mass? Choose the words and numbers to complete the statements.Convert the masses of both fruits to the same unit.85,000 milligrams is the same asChoose...grams.0.15 kilograms is the same asChoose...grams.TheChoose...has the greater mass. How many triangles exist that fit the following criteria?B = 30, a = 4, b = 3 What is the product?-1/5[-2 1/4] Ignore the second part where it has a question mark that was my attempt the question is on top. Jacques, c'est ________ ami de Yolande Find the compound interest and future value. Round your answers to the nearest cent. Do not round intermediate steps.Principal Rate Compounded Time$580 4.65% Quarterly6 yearsThe future value is $and the compound interest is $SCheck what was the similarity and difference for women roles in north america and central america from 1450-1700 if x + 9 equals 13 what is the value of x Which one is a true statement about exothermic reactions?A) Energy from outside is added.B) The reaction always feels cold.C) Heat is a reactant.D) Energy is released as a product. message writing(help me plss A market analyst has projected that the cost of producing d dog leashes will be given by the polynomial10,500 + 3,9d, The revenue generated from the sale of d dog leashes will be given by the polynomial.d14,5 - 0,000030), Find a polynomial expression for the profit earned from producing and selling d dogleashes, Evaluate the expression for da 30,000. $514,500 $295,650 $280,500 $535,500 The triangle is a 45-45-90 triangle.x2 = 6636X =(2)x = 18Did not multiply the numerator by the radicalShould have squared the denominator only.No ErrorShould have squared the numerator onlyJUN1O6V2=82 Ceasia needs a new pair of basketball shoes for the upcoming season. The price of the shoes she wants is $98.99. The tax on the shoes is 5.2%. Whatis the exact price Ceasia will pay for her shoes tax included? 9. When Mei stopped 80% of the shots ongoal in the last game, she stopped 16 shots.How many goals were scored? Question 1Refer to your Who Is Sonia Sotomayor? book for a complete version of this text.Part AWhat is a main idea of Chapter 7?A. Sonia has trouble choosing between working as a judge or running for public office. B. Sonia earns a reputation as a fair and capable judge with her ruling in the baseball strike case.C. As judge with experience, Sonia requires little time to consider both sides of a case before ruling.D. Despite all her experience, Sonia finds she is uncomfortable in her role as a judge.Part B Which detail from the text best supports the answer to Part A?A. Some people thought Judge Sotomayor would take days or weeks to decide the case.B. He asked the mayor of New York to put Sonia on a committee that looked at how money was spent in political campaigns.C. She was so nervous, her knees knocked together.D. That meant she didnt need any spring trainingor practiceto do her job well.